You are on page 1of 81

CLASSIFICATION OF COST

Manufacturing and non-manufacturing costs


Costs may be classified as manufacturing costs and non-manufacturing costs. This classification is
usually used by manufacturing companies.

Manufacturing costs:
Manufacturing costs can be further divided into the following categories:
1. Direct materials
2. Direct labor
3. Manufacturing overhead

The above three categories of manufacturing costs are briefly explained below:

Direct materials:
Materials that become an integral part of the finished product and that can be easily traced to it are called
direct materials. For example wood is a direct material for the manufacturers of furniture. Lime stone is
direct material for the manufacturers of cement. Direct materials usually consists of a significant portion of
total manufacturing cost. Direct material is sometime called raw material.
The finished product of a company may become raw material of another company. For example, cement
is a finished product for manufacturers of cement and raw materials for companies involved in
construction business.

Direct labor:
The labor cost that can be physically and conveniently traced to a unit of finished product is called direct
labor cost or touch labor cost. Examples of direct labor cost include labor cost of machine operators
and painters in a manufacturing company. Like direct materials, it comprises of a significant portion of
total manufacturing cost.
The sum of direct materials cost and direct labor cost is known as prime cost.
Formula of prime cost:
Prime cost = Direct materials cost + Direct labor cost

Manufacturing overhead cost:


Manufacturing costs other than direct materials and direct labor are categorized as manufacturing
overhead cost (also known as factory overhead costs). It usually includes indirect materials, indirect labor,
salary of supervisor, lighting, heat and insurance cost of factory etc. Usually, manufacturing overhead
costs cannot be easily traced to individual units of finished products.
The sum of direct labor cost and manufacturing overhead cost is known as conversion cost.

Formula of conversion cost:


Conversion cost = Direct labor cost + Manufacturing overhead cost
The sum of direct materials cost, direct labor cost and manufacturing overhead cost is known
as manufacturing cost.

Formula of manufacturing cost:


Manufacturing cost = Direct materials cost + Direct labor cost + Manufacturing overhead cost

Non-manufacturing costs:
Non-manufacturing costs are further divided into the following categories:
1. Marketing and selling costs
2. Administrative costs
Examples of marketing and selling costs include advertising costs, order taking costs and salaries of
sales persons etc. Examples of administrative costs include salaries of executives, accounting costs, and
general administration costs etc.

Product costs and period costs

Costs may be classified as product costs and period costs. This classification is usually used for financial
accounting purposes. A brief explanation of product costs and period costs is given below:

Product costs:
Product costs (also known as inventoriable costs) are those costs that are incurred to acquire or
manufacture a product. For a manufacturing company, theses costs usually consist of direct materials,
direct labor, and manufacturing overhead.

Product costs are initially treated as inventory and do not appear on income statement until the product
for which they are incurred is sold. When the product is sold, these costs are transferred to cost of goods
sold account. For example, if a company manufactures 50 units of product X and sells only 30 units in
2013. The direct materials, direct labor and manufacturing overhead costs incurred to manufacture these
50 units will be initially treated as inventory (an asset). The inventory of 30 units will be transferred to cost
of goods sold during the year 2013 and appear on the income statement of 2013. The remaining
inventory of 20 units will not be transferred to cost of good sold in 2013.

Period costs:
The costs that are not included in product costs are known as period costs. Usually, these costs are not
part of the manufacturing process and are therefore treated as expense for the period in which they arise.
Period costs are not attached to products and company does not need to wait for the sale of products to
recognize them as expense. According to generally accepted accounting principles (GAAP), all
marketing, selling and administration costs are treated as period costs. Examples of these costs include
office rent, interest, depreciation of office building, sales commission and advertising expenses etc.
A summary of the concepts explained above is given below:

Variable, fixed and mixed (semi-variable) costs


As the level of business activities changes, some costs change while others do not. The response of a
cost to a change in business activity is known as cost behavior. Managers should be able to predict the
behavior of a particular cost in response to a change in particular business activity. For this purpose,
costs are classified as variable, fixed and mixed costs. This article explains these three types
of costs as well as their response to business activities.

Variable cost:
A cost that changes, in total dollar amount, with the change in the level of activity is called variable cost.
A common example of variable cost is direct materials cost. Consider the following example to
understand how variable cost behaves in a manufacturing company.
A mobile phone manufacturing company purchases speakers from another company at a cost of $2 per
speaker. The speaker is a direct materials cost for mobile phone manufacturing company. One speaker is
used to complete a mobile phone. The total and per unit cost of speakers at various levels of activity is
given below:
Notice that the total cost of speakers increases as the mobile phones produced are increased but per unit
cost remains constant.
Other examples of variable cost include lubricants, sales commission and shipping costs etc.
Fixed cost:
A cost that does not change, in total, with the change in activity is called fixed cost. A common example
of fixed cost is rent. In above example, if mobile phone manufacturing company rents a building for its
factory for $5,000 per month, it will have to pay $5,000 for every month even no mobile phone is
produced during the month. The behavior of fixed is shown in the following figure:

Total fixed cost does not change with the change in activity but per unit fixed cost changes with the rise
and fall in the level of activity. There is an inverse relationship between per unit fixed cost and activity. If
production increases, per unit fixed cost decreases and if production decreases, per unit fixed cost
increases. To understand this point, we can use the data from the above example
of mobile manufacturing company. Consider the following table:

Notice that average fixed cost (computed in the last column) decreases as the production of mobile
phones increases. It is an interesting property of fixed cost.
Mixed or semi-variable cost:
A cost that has the characteristics of both variable and fixed cost is called mixed or semi-variable cost.
For example, the rental charges of a machine might include $500 per month plus $5 per hour of use. The
$500 per month is a fixed cost and $5 per hour is a variable cost. Another example of mixed or semi-
variable cost is electricity bill. The electricity bill can be divided into two parts – (1) line rent and (2) cost of
units consumed. The line rent is not affected by the consumption of electricity whereas the cost of units
consumed varies with the change in units consumed.

Direct costs and indirect costs


Costs may be classified as direct costs or indirect costs. The purpose of this classification is to
assign costs to cost objects. Cost object means any thing about which cost information is collected. Some
examples of cost objects are products, departments, customers, plant, a territory, a product line
and research and development activities of the business etc.
A brief explanation of direct costs and indirect costs are given below:

Direct cost:
A cost that is easily traceable to a particular cost object is known as direct cost. The use of the term
“direct cost” is not limited to direct materials and direct labor. Every cost that can be easily and
conveniently traced to a particular product, customer, branch, plant or any other cost object is a direct
cost. For example, if Bata shoes company wants to assign costs to all the branches in Pakistan, then the
salary of the manager of Islamabad branch would be a direct cost of that branch.

Indirect cost:
A cost that is not easily traceable to a particular cost object is called indirect cost. For example, a clothing
factory produces different varieties of cloths. The salary of the manager would be an indirect cost
because it is caused by all the varieties and is not easily traceable to a particular variety. The Rafhan
Maize Products company produces a large number of products by processing tones of corn every year.
The salary of the factory manager is an indirect cost for the products because it is not caused by a
particular product.
The costs that are caused by a number of cost objects but cannot be traced to a particular cost object is
known as common cost. In our examples, the salaries of the managers of clothing factory and Rafhan
maize products are common costs.
When we say direct or indirect cost, we mean that it is direct or indirect with respect to a particular cost
object. A cost may be direct for one cost object but indirect for another. For example, National Food
Products Co. has a number of branches in Pakistan. Each branch sell a variety of food products. The
salary of the manager of Karachi branch would be an indirect cost of a particular product but direct cost of
the branch as a whole.
Differential, opportunity and sunk costs
Costs may be classified as differential cost, opportunity cost and sunk cost. This classification is
made for decision making purposes. Explanation and examples of differential, opportunity and
sunk costs are given below:

Differential cost:
The work of managers includes comparison of costs and revenues of different alternatives. Differential
cost (also known as incremental cost) is the difference in cost of two alternatives. For example, if the
cost of alternative A is $10,000 per year and the cost of alternative B is $8,000 per year. The difference of
$2,000 would be differential cost. The differential cost can be a fixed cost or variable cost.
Similarly the difference in revenue of two alternatives is known as differential revenue. For example, if
alternative A’s revenue is $15,000 and alternative B’s revenue is $10,000. The difference of $5,000 would
be differential revenue.
When different revenue generating alternatives are compared, the differential cost as well as differential
revenues associated with each alternative is taken into account.
The terms “differential cost” and “differential revenue” used in managerial accounting are similar to the
terms “marginal cost” and “marginal revenue” used in economics.
Example – computation of differential cost, differential revenues and differential
net operating income:
The management of Galaxy company has two alternatives to choose from. Compute differential revenue,
differential cost and differential net operating income from the information of two alternatives given below:
In the above example, total differential revenue is $200,000 (1,600,000 – 1,400,000), differential cost is
$130,000 (1,240,000 – 1,110,000) and differential net operating income is $70,000 ($360,000 –
$290,000).
If a decision is made on the basis of above computations, alternative 2 would be selected because it
promises to generate more net operating income.

Opportunity cost:
Unlike other types of cost, opportunity cost does not require the payment of cash or its equivalent. It is a
potential benefit or income that is given up as a result of selecting an alternative over another. For
example, You have a job in a company that pays you $25,000 per year. For a better future, you want to
get a Master’s degree but cannot continue your job while studying. If you decide to give up your job and
return to school to earn a Master’s degree, you would not receive $25,000. Your opportunity cost would
be $25,000.
Almost every alternative has an opportunity cost. It is not entered in the accounting records but must be
considered while making decisions.
Sunk cost:
The costs that have already been incurred and cannot be changed by any decision are known as sunk
costs. For example, a company purchased a machine several years ago. Due to change in fashion in
several years, the products produced by the machine cannot be sold to customers. Therefore the
machine is now useless or obsolete. The price originally paid to purchase the machine cannot be
recovered by any action and is therefore a sunk cost.
These costs should not be taken into account while making any decision because no action can reverse
them.

Costs of quality or quality costs


Costs of quality or quality costs does not mean the use of expensive or very highly quality materials to
manufacture a product. The term refers to the costs that are incurred to prevent, detect and remove
defects from products. Quality costs are categorized into four main types. Theses are:
1. Prevention costs
2. Appraisal costs
3. Internal failure costs and
4. External failure costs.

These four types of quality costs are briefly explained below:


Prevention costs:
It is much better to prevent defects rather than finding and removing them from products.
The costs incurred to avoid or minimize the number of defects at first place are known as prevention
costs. Some examples of prevention costs are improvement of manufacturing processes,
workers training, quality engineering, statistical process control etc.
Appraisal costs:
Appraisal costs (also known as inspection costs) are those cost that are incurred to identify defective
products before they are shipped to customers. All costs associated with the activities that are performed
during manufacturing processes to ensure required quality standards are also included in this category.
Identification of defective products involve the maintaining a team of inspectors. It may be very costly for
some organizations.

Internal failure costs:


Internal failure costs are those costs that are incurred to remove defects from the products before
shipping them to customers. Examples of internal failure costs include cost of rework, rejected products,
scrap etc.
External failure costs:
If defective products have been shipped to customers, external failure costs arise. External failure costs
include warranties, replacements, lost sales because of bad reputation, payment for damages arising
from the use of defective products etc. The shipment of defective products can dissatisfy customers,
damage goodwill and reduce sales and profits.

More examples of quality costs


Examples of prevention cost
 System development
 Quality engineering
 Quality training
 Quality circles
 Statistical process control
 Supervision of prevention
 Quality improvement projects
 Technical support to suppliers
 Quality data gathering, analysis and reporting
 Audit of the quality system

Examples of appraisal cost


 Test and inspection of incoming materials
 Final product testing and inspection
 Supplies used in testing and inspection
 Supervision of testing and inspecting activities
 Depreciation of test equipment
 Maintenance of test equipment
 Plant utilities in inspection area
 Field testing and appraisal at customer site

Examples of internal failure cost


 Net cost of scrap
 Net cost of spoilage
 Rework labor and overhead
 Reinspection of reworked products
 Disposal of defective products
 Down time caused by quality problems
 Analysis of the cause of defects in the production
 Retesting of reworked products
 Re-entering data because of keying
 Debugging software errors
Examples of external failure cost
 Cost of field servicing and handling complaints
 Warranty repairs and replacement costs
 Liability arising from defective products
 Lost sales arising from a reputation of poor quality
 Returns and allowances arising from quality problems
 Product recalls
 Repairs and replacements beyond the warranty period

Treatment of idle time, overtime premium, and fringe benefit costs


In addition to direct labor costs, there are other costs associated with direct labor workers. These are idle
time, overtime premium and fringe benefits that are provided to workers. These are not part of direct labor
cost. The following paragraphs explain the computation and accounting treatment of idle time, overtime
premium and labor fringe benefits:

Treatment of idle time:


Idle time means the amount of time the workers remain idle in a normal working day. The idle time is
usually caused by a sudden fault in machine or equipment, power failure, lack of orders for the product,
inefficient work scheduling, defective materials and shortage of raw materials etc. The cost associated
with idle time is treated as indirect labor cost and should, therefore, be included in manufacturing
overhead cost. For example, the normal weekly working hours of a worker are 48 and he is paid @ $8 per
hour. If he remains idle for 6 hours due to power failure, then the cost of 42 hours would be treated as
direct labor cost and the cost of 6 hours (idle time) would be treated as indirect labor cost and included in
manufacturing overhead cost.

Treatment of overtime premium:


Overtime premium is the amount that is paid, for the overtime worked, in excess of the normal wage rate.
Like idle time, overtime premium is also treated as indirect labor cost and included in manufacturing
overhead cost. For example, a worker normally works for 48 hours per week @ $8 per hour. In a
particular week, if he works for 52 hours and company pays him $12 for every hour worked in excess of
48 hours, the allocation of the labor cost of the worker would be made as follows:

The amount of $16 is overtime premium and is a part of manufacturing overhead cost.
Treatment of labor fringe benefits:
Fringe benefits are benefits that employers provide to employees in addition to normal salaries or wages.
Examples of fringe benefits are hospitalization, insurance programs, retirement plans, paid holidays and
stock options etc. Most of the companies treat labor fringe benefits as indirect labor and, therefore,
include them in manufacturing overhead costs.
A few firms treat direct labor related fringe benefits as addition direct labor cost that is considered a more
superior practice.
The above information has been summarized below:

High-low point method


High-low point method is a technique used to divide a mixed cost into its variable and fixed
components.
Sometimes it is necessary to determine the fixed and variable components of a mixed cost figure. Several
techniques are used for this purpose such as scatter-graph method, least squares method and high-low
point method. On this page I will explain the use of high-low point method.
Under high-low point method, an estimated variable cost rate is calculated first using the highest and
lowest activity levels and mixed costs associated with them. This estimated variable cost rate is used to
calculate total estimated variable cost included in the mixed cost figures at highest and lowest activity
levels. The estimated variable cost is then subtracted from the total mixed cost figures at highest and
lowest activity levels to find the fixed cost component.
To make the procedure simple and easy to understand , we can divide the calculations into the following
three steps.

Step 1 – calculation of variable cost rate:


The first step in high-low point method is to calculate an estimated variable cost rate. This rate is
calculated by using the following formula:
Step 2 – calculation of variable cost component:
After calculating estimated variable cost rate, the second step is to calculate the total estimated variable
cost at highest and lowest activity levels. It is calculated by multiplying the estimated variable cost rate
(calculated in step 1) by highest and lowest activity levels. The formula is given below:
Estimated total variable cost = Estimated variable cost rate × Highest or lowest level of activity

Step 3 – calculation of fixed cost component:


The third and final step in high-low point method is to find out the fixed cost component of the total mixed
cost. It is calculated by subtracting the estimated variable cost (calculated in step 2) from the total mixed
cost figure. The formula for this purpose is given below:
Fixed cost = Total mixed cost – Estimated total variable
Example:
The Western company presents the production and cost data for the first six months of the 2015.

Required: Determine the estimated variable cost rate and fixed cost using high-low point method.
Solution:
Variable cost rate
(66,000 – 45,000)/(29,000 – 15,000)
= $21000/14,000
= $1.5 per unit
Variable cost:
 Highest activity level (May): 29,000 units × $1.5 = $43,500
 Lowest activity level (January): 15,000 × $1.5 = $22,500
Fixed cost:
 Highest activity level (May): $66,000 – $43,500 = $22,500
 Lowest activity level (January): $45,000 – $22,500 = $22,500
Variable costing versus absorption costing
Explain the difference between variable and absorption costing. How unit product cost is computed under
two methods?
Variable and absorption are two different costing methods. Almost all successful companies in the world
use both the methods. Variable costing and absorption costing cannot be substituted for one another
because both the systems have their own benefits and limitations.
These costing approaches are known by various names. For example, variable costing is also known as
direct costing or marginal costing and absorption costing is also known as full costing or traditional
costing.
The information provided by variable costing method is mostly used by internal management for decision
making purposes. Absorption costing provides information that is used by internal management as well as
by external parties like creditors, government agencies and auditors etc.

Computation of unit product cost under two methods:


Under absorption costing system, the product cost consists of all variable as well as all
fixed manufacturing costs i.e., direct materials, direct labor and factory overhead (FOH). But when
variable costing system is used, the fixed cost (both manufacturing and non-manufacturing) is treated as
a period or capacity cost and is, therefore, not included in the product cost.
Following exhibition summarizes the difference between variable costing and absorption costing:
Variable versus absorption costing

For further clarification of the concept, consider the following examples:


Example 1
A company manufactures and sells 5000 units of product X per year . Suppose one unit of product X
requires the following costs:
Direct materials: $5 per unit
Direct labor: $4 per unit
Variable manufacturing overhead: $1 per unit
Fixed manufacturing overhead: $20,000 per year
The unit product cost of the company is computed as follows:
Absorption Costing Variable Costing

$5 $5

$4 $4

$1 $1

$4* –

——- ——-

$14 $10

——- ——-

* $20,000 / 5,000
Notice that the fixed manufacturing overhead cost has not been included in the unit cost under variable
costing system but it has been included in the unit cost under absorption costing system. This is the
primary difference between variable and absorption costing.
Example 2
Sunshine company produces and sells only washing machines. The company uses variable costing for
internal reporting and absorption costing for external reporting. The data for the year 2010 is given
below:
Direct materials $150/unit

Direct labor $45/unit

variable manufacturing overhead $25/unit

Fixed manufacturing overhead $160,000 per year

Fixed marketing and administrative expenses $110,000 per year

Variable marketing and administrative expenses $15/unit sold

Company produced and sold 8,000 machines during the year 2010.
Required: Compute unite product cost under variable costing and absorption costing.

Solution
Absorption Costing Variable Costing

Materials $150 $150

Labor $45 $45

Variable overhead $25 $25

Fixed overhead $20* —

——- ——-

240 220

——- ——-

*$160,000 / 8,000 Units = $20


Note: Marketing and administrative expenses are period costs and are not relevant in the computation of
unit product cost.

Causes of difference in net operating income under variable and


absorption costing
Why variable costing and absorption costing produce different net operating income? How to reconcile
net operating income figures produced by these two costing approaches?
Variable costing and absorption costing usually produce different net operating income figures. The
reason is that the fixed manufacturing overhead cost is not treated the same way under two costing
methods. To understand how the difference in treatment of fixed manufacturing overhead cost changes
the net operating income figures of two costing systems, we need to prepare two income statements, one
under variable costing and one under absorption costing. For this purpose, consider the following
example:

Example
A company prepares variable costing income statement for the use of internal management and
absorption costing income statement for the use of external parties like creditors, banks, tax authorities
etc. The company manufactures a product that is sold for $80. The variable and fixed cost data is given
below:
Direct materials $30.00

Direct labor $19.00

Factory over head:

Variable cost $6.00

Fixed cost ($45,000 / 9000 units) $5.00

Marketing, general and administrative:

Variable cost (per unit sold) $4.00

Fixed cost (per month) $28,000

During June 9,000 units were produced and 7,500 units were sold. The opening inventory was 2,000
units.
Required:
1. Prepare two income statements, one using variable costing method and one using absorption costing
method.

2. Explain the difference in net operating income (if any) under two approaches.
Solution
(1)
Income Statement – Absorption Costing
Sales (7,500 units × $80) $600,000

Cost of goods sold:

Beginning inventory (2000 units × $60) $120,000

Units manufactured this month (9,000 × $60) $540,00

———-

Available for sale $660,000


Ending inventory (3,500* × $60) $210,000

———-

Cost of goods sold $450,000

———-

Gross profit $150,000

Marketing, general and administration expenses:

Variable (7,500 units × $4) $30,000

Fixed $28,000 $58,000

———- ———-

Net operating income $92,000

———-

*Computation of units in ending inventory:


Beginning inventory 2,000 units

Produced during the month 9,000 units

————

Units available for sale 11,000 units

Sold during the month 7,500 units

————

Ending inventory 3,500 units

————-

Income Statement – Variable Costing


Sales (7,500 × $80) $600,000

Variable cost of goods sold:

Beginning inventory (2000 units × $55) $110,000

Units manufactured this month (9,000 units × $55) $495,00

———-

Available for sale $605,000


Ending inventory (3,500 units × $55) $192,500

———-

Variable cost of goods sold $412,500

———-

Gross contribution margin $187,500

Variable marketing, general and administration expenses: $30,000

———-

Contribution margin $157,500

Less fixed costs:

Manufacturing overhead $45,000

Marketing, general and administration expenses $28,000 $73,000

———- ———-

Net operating income $84,500

———-

(2) Reconciliation of net operating income:


Net operating income under variable costing $84,500

Fixed manufacturing overhead cost deferred (1500 units × $5) $7,500

——–

Net operating income under absorption costing $92,000

——–

Explanation of the difference in net operating income:


Notice that the net operating income under absorption costing is $7,500 ($92,000 – $84,500) higher than
the net operating income under variable costing. This difference is because of fixed manufacturing
overhead that becomes the part of ending inventory under absorption costing system. The ending
inventory absorbs a portion of fixed manufacturing overhead and reduces the burden of the current
period. In this way a portion of fixed cost that relates to the current period is transferred to the next period.
Under variable costing, the fixed manufacturing overhead cost is not included in the product cost but
charged to the income statement of the relevant period in its entirety. Therefore no portion of fixed cost is
absorbed by the ending inventory.
In our example, the net operating income is higher under absorption costing than variable costing
because closing inventory is higher than the opening inventory.
Important points to remember:
1. The net operating income under absorption costing systems is always higher than variable costing
system when inventory increases.
2. The net operating income under variable costing systems is always higher than absorption costing
system when inventory decreases.
3. When inventory increases, the fixed manufacturing overhead cost is deferred to inventory.
4. When inventory decreases, the fixed manufacturing overhead cost is released from inventory.

Variable and absorption costing with just in time (JIT)


manufacturing system
Explain the impact of just in time (JIT) manufacturing and inventory control system on the variable and
absorption costing income statements of the companies.
A company using both variable and absorption costing usually finds a difference in net operating income
figures produced by the income statements prepared under these two costing methods. This difference
may be confusing especially for those who do not fully understand these two costing approaches.
The difference in the net operating income under variable and absorption costing can largely be reduced
by using a system called just in time (JIT) manufacturing and inventory control system. This system
encourages companies to eliminate all types of inventory (materials, work-in-process and finished goods).
When inventories are reduced, the difference in the net operating income figure is automatically reduced.
Adoption of just in time (JIT) does not change the computation of unit product cost but eliminates the
inventories. When inventories are eliminated, the difference in net operating income is automatically
eliminated because they are the root cause of the difference.
Example:
The following data relates to two manufacturing companies – company A and company B. Company A
uses traditional manufacturing system and company B uses a strict just in time (JIT) manufacturing
system. Company B does not manufacture a unit unless an order is received for it.
Company A Company B

Sales price per unit $40 $60

Manufacturing expenses:

Direct materials $15 $15

Labor $5 $10

Factory overhead:

Variable $2 $10

Fixed – (A: 7,500/m B: $9,375/m ) $3 $5

——- ——-

Total manufacturing expenses $25 $40

——- ——-

Opening inventory 500 Units 0

Production 2,500 Units 1875 Units

————– ————–

Available for sale 3,000 Units 1875 Units

Closing inventory 300 Units 0

————– ————–

Sales 2,700 Units 1,875 Units

————– ————–

Marketing and admin expenses:

Variable (per unit sold) $5 $8

Fixed $5000 $4,000

Now we will prepare income statements of both the companies under variable costing and absorption
costing methods and observe the impact of just in time (JIT) manufacturing system on the company B’s
net operating income figure.
Absorption costing:
Company A Company B

Sales revenue 108,000 112,500

———- ———-

Less cost of goods sold (COGS):

Opening inventory 12,500 0

Cost of goods manufactured 62,500 75,000

———- ———-

Cost of goods available for sale 75,000 75,000

Closing inventory 7,500 0

———- ———-

Cost of goods sold (COGS) 67,500 75,000

———- ———-

Gross margin (sales – COGS) 40,500 37,500

———- ———-

Less marketing and administration exp:

Variable 13,500 15,000

Fixed 5,000 4,000

———- ———-

Total marketing and administration exp. 18,500 19,000

———- ———-

Net operating income 22,000 18,500

———- ———-

Variable costing:
Company A Company B

Sales revenue 108,000 112,500

———- ———-
Less variable cost of goods sold (VCOGS):

Opening inventory 11000 0

Cost of goods manufactured 55,000 65,625

———- ———-

Cost of goods available for sale 66,000 65,625

Closing inventory 6,600 0

———- ———-

Variable cost of goods sold (VCOGS) 59,400 65,625

———- ———-

Gross contribution margin (Sales – VCOGS) 48,600 46,875

Less variable marketing and admin. exp 13,500 15,000

———- ———-

Contribution margin 35,100 31,875

———- ———-

Less fixed expenses:

Manufacturing 7,500 9,375

Marketing and administration 5,000 4,000

———- ———-

Total fixed expenses 12,500 13,375

———- ———-

Net operating income 22,600 18,500

———- ———-

Company A’s net operating income is different under two costing methods because it does not follow just
in time (JIT) system (maintains inventory).
Company B’s net operating income is same under both the costing methods because it follows just in
time (JIT) system (does not maintain inventory).
The change in inventory during the period is responsible for the difference in net operating income.
Company B does not maintain any inventory hence no change in inventory. When inventory does not
change the operating income remains same under variable costing and absorption costing.
In our example company B’s opening and closing inventory is zero, but in practice it may not always be
possible. Companies using just in time method may have some opening and closing inventories. The
concept of just in time is to maintain minimum inventory. When inventories are minimized, confusion of
operating income difference between variable and absorption costing is automatically minimized.

Important points to remember:


1. The change in inventory is the reason of difference in the net operating income figure under variable
costing and absorption costing.
2. The difference in the net operating income figure can be minimized or eliminated by adopting just in time
manufacturing system.

Advantages and disadvantages of variable costing


What are advantages and disadvantages of variable costing system?
Companies need absorption costing to prepare statements to satisfy external parties and variable costing
for better management. Both the costing methods have benefits and limitations. Following are the main
advantages and disadvantages of variable costing system:

Advantages
1. Variable costing provides a better understanding of the effect of fixed costs on the net profits because
total fixed cost for the period is shown on the income statement.
2. Various methods of controlling costs such as standard costing system and flexible budgets have close
relation with the variable costing system. Understanding variable costing system makes the use of those
methods easy.
3. Companies using variable costing system prepare income statement in contribution margin format that
provides necessary information for cost volume profit (CVP) analysis. This data cannot be directly
obtained from a traditional income statement prepared under absorption costing system.
4. The net operating income figure produced by variable costing is usually close to the flow of cash. It is
useful for businesses with a problem of cash flows.
5. Under absorption costing system, income of different periods changes with the change of inventory
levels. Sometime income and sales move in opposite directions. But it does not happen under variable
costing.
Disadvantages
1. Financial statements prepared under variable costing method do not conform to generally accepted
accounting principles (GAAP). The auditors may refuse to accept them.
2. Tax laws of various countries require the use of absorption costing.
3. Variable costing does not assign fixed cost to units of products. So the production costs cannot be truly
matched with revenues.
4. Absorption costing is usually the base for evaluating top executive’s efficiency.
EXERCISES

Super Bike Manufacturing Company presents the following data for 2011:

Opening inventory 0 Units


Sales 8,000 Units
Production 10,000 Units
Closing inventory 2,000 Units
Direct materials $240
Direct labor $280
Variable manufacturing overhead expenses $100
Variable selling and administrative expenses $40
Fixed manufacturing overhead expenses $1200,000
Fixed selling and administrative expenses $800,000
Required: Compute the unit product cost of one bike under:
1. Absorption costing system.
2. Variable costing system.

Solution:
Computation of unit product cost:
Absorption costing Variable costing
Direct materials $240 $240
Direct labor $280 $280
Variable manufacturing overhead $100 $100
Fixed manufacturing overhead $120* —
———- ———-
Unit product cost $740 $620
———- ———-
*1,200,000 / 10,000 = $120
Notice that the fixed manufacturing overhead cost has not been included while computing the cost of one
bike under variable costing system.
Note: Selling and administrative expenses (both variable and fixed) are not relevant for the computation
of unit product cost.

Exercise-2 (Variable costing income statement, Reconciliation of net


operating income)
The following is the absorption costing income statement of a manufacturing company:
Sales (40,000 units @ $67.50) $2,700,000

Less cost of goods sold:

Opening inventory 0
Add cost of goods manufactured (50,000 × 42) 2,100,000

———-

Available for use 2,100,000

Less closing inventory 420,000 1,680,000

———- ———-

Gross margin 1,020,000

Less selling and administrative expenses 840,000

———-

Net operating income 180,000

———-

Fixed selling and administrative expenses are $600,000. Variable selling and administrative expenses
are $6 per unit sold. The unit product cost under absorption costing is computed as follows:

Direct materials $20

Direct labor 8

Variable manufacturing overhead 4

Fixed manufacturing overhead ($500,000/50,000) 10

——-

Total cost per unit $42

——-

Required:
1. Prepare a contribution margin income statement using variable costing system.
2. Reconcile any difference between net operating income figure under variable costing income statement
and net operating income figure under absorption costing income statement.
Solution
(1) Income Statement:
Sales (40,000 units @ $67.50) 2,700,000

Less variable cost of goods sold:

Opening inventory 0

Add v. cost of goods manufactured (50,000 units × $32) 1,600,000


———-

Available for sale 1,600,000

Less closing inventory (10,000 units × $32) 320,000 1,280,000

———- ———-

Gross contribution margin 1,420,000

Less variable selling and administrative expenses 240,000

———-

Contribution margin 1,180000

Less fixed expenses:

Manufacturing 500,000

Selling and admin. 600,000 1,100,000

———- ———-

Net operating income 80,000

———-

(2) Reconciliation of net operating income:


Net operating income under variable costing 80,000

Fixed manufacturing overhead deferred in inventory (10,000 units × $10) 100,000

——–

Net operating income under absorption costing 180,000

——–

Exercise-3 (Unit product cost under variable costing, break-even


point)
A company manufactures and sells large size tables to be used in the offices of the executives. One
table is sold for $400. The data for 2010 is as follows:
Manufacturing costs:

Direct materials per unit $120

Direct labor per unit $60


Variable manufacturing overhead per unit $20

Fixed manufacturing overhead per year $600,000

Non-manufacturing costs:

Variable selling administrative per unit $40

Fixed selling administrative $900,000

Inventory:

Opening 0

Production during 2010 10,000

———

Units available for sale 10,000

Sales 9,000

———

Closing inventory 1,000

———

Required:
1. Compute cost of one table under variable costing.
2. Prepare income statement if variable costing is used.
3. Compute breakeven point in units.
4. Calculate net operating income of the company under absorption costing by preparing a reconciliation
schedule.
Solution
(1) Computation of the cost of one office table:
Direct materials $120

Direct labor $60

Variable manufacturing over head $20

——

Cost of one table under variable costing $200

——

(2) Income statement


Sales (9,000 units @ $400) 3,600,000

Less variable cost of goods sold:

Opening inventory 0

Add v. cost of goods manufactured (10,000 units × $200) 2,000,000

———-

Available for sale 2,000,000

Less closing inventory (1,000 units × $200) 200,000 1,800,000

———- ———-

Gross contribution margin 1,800,000

Less variable selling and administrative expenses 360,000

———-

Contribution margin 1,440,000

Less fixed expenses:

Manufacturing 600,000

Selling and admin. 900,000 1,500,000

———- ———-

Net operating loss (60,000)

———-

(3). Break-even point:


We can easily calculate break-even point using contribution margin method:
= Total fixed expenses / Contribution margin per unit
= $1,500,000 / $160*
= 9,375 Units
*$1,440,000 / 9,000 Units
(4) Net operating income under absorption costing:
Net operating income (loss) if variable costing is used $(60,000)

Fixed manufacturing overhead cost deferred in inventory (1000 units × $60*) $60,000

———-

Net operating income if absorption costing is used 0


———-

*$600,000/10,000 units = $60

Exercise-4 (Variable and absorption costing ending inventory,


external reports)
The production and sales data of Albari company for the year 2011 is as follows:
Variable costs per unit:

Direct materials $20

Direct labor $10

Variable manufacturing overhead $4

Variable selling and administrative expenses $8

Fixed costs per year:

Fixed manufacturing overhead $180,000

Fixed selling and administrative expenses $600,000

During 2011, Albari company manufactured 30,000 units out of which 25,000 units were sold. At the end
of 2011, the finished goods inventory account showed a balance of $170,000.
Required:
1. What costing method is used by Albari to compute finished goods inventory?
2. Should company use $170,000 finished goods inventory figure for external reports? if not what is correct
amount in dollars that the company should use for external reporting purpose?
Solution:
(1) Costing method used:
The ending inventory figure of $170,000 shows that the company is using variable costing for finished
goods inventory because the company has not included fixed manufacturing cost in its ending inventory.
The following calculation proves that:
Variable cost per unit:
Direct materials $20

Direct labor $10

Variable manufacturing overhead $4

——–

Total variable cost per unit $34

——–

Ending inventory under variable costing:


$34 × 5,000 = $170,000

(2) Finished goods inventory figure for external reports:


For external reporting purposes, company must use finished goods inventory figure computed on the
basis of absorption costing system. It is computed as follows:
Absorption cost per unit:
Direct materials $20

Direct labor $10

Variable manufacturing overhead $4

Fixed manufacturing overhead $6*

——–

Total absorption cost per unit $40

——–

Ending inventory under absorption costing:

$40 × 5,000 = $200,000

* $180,000/30,000 units = $6.00


Exercise-5 (Variable and absorption costing income statement,
reconciliation)
AGA company manufactures and sells a product for $20/Kg. The data for the last year is given below:
Sales 75,000 Kg

Finished goods inventory at the beginning of the period 12,000 Kg

Finished goods inventory at the closing of the period 17,000 Kg

Manufacturing costs:

Variable cost $8 per Kg

Fixed manufacturing overhead cost $320,000 per year

Marketing and administrative expenses:

Variable expenses $2 per Kg of sale

Fixed expenses $300,000 per year

Required:
1. Income statement using absorption and variable costing methods.
2. Explanation of the cause of difference in operating income under two concepts.
Solution
(1) Income statements:
(a) Absorption costing:
Sales 1,500,000

Less cost of goods sold:

Beginning inventory (12000Kg × $12) 144,000

Cost of goods manufactured (80,000* Kg × $12**) 960,000

———

Cost of goods available for sale 1,104,000

Closing inventory (17,000Kg × 12 ) 204,000 900,000

——— ———

Gross profit 600,000

Less marketing and administrative expenses:

Variable expenses (75,000Kg × $2) 150,000

Fixed expenses 300,000 450,000

——— ———

Net operating income 150,000

———

*Production for last year: **Manufacturing cost per unit

Sales 75,000Kg Variable $8

Closing inventory 17,000Kg Fixed (320,000 / 80,000) $4

———- ——

Total inventory available for sale 92,000Kg $12

Opening inventory 12,000Kg ——

———-

Production for the period 80,000Kg

———-

(b)Variable costing:
Sales 1,500,000

Less variable cost of goods sold:

Beginning inventory (12000Kg × $8) 96,000

Variable cost of goods manufactured (80,000 Kg × $8) 640,000

———

Variable cost of goods available for sale 736,000

Closing inventory (17,000Kg × 8 ) 136,000 600,000

——— ———

Gross contribution margin 900,000

Variable marketing and admin. expenses (75,000Kg × $2) 150,000

———

750,000

Less period costs:

Marketing and administrative 300,000

Manufacturing 320,000 620,000

——— ———

Net operating income 130,000

———

(2) Explanation of the difference in net operating income:


The net operating income under absorption costing is $20,000 more than the net operating income under
variable costing. When production is more than sales (as in this exercise), the fixed manufacturing
overhead is deferred in inventory that causes a higher net operating income under absorption costing
than under variable costing. The reconciliation of net operating income is as follows:
Operating income under absorption costing 150,000

Operating income under variable costing 130,000

———

Difference in net operating income 20,000


———

Change in inventory (17,000Kg – 12,000Kg) 5,000Kg

———

Fixed cost deferred in inventory (5,000Kg × $4.00) 20,000

———

or
Net operating income under variable costing 130,000

Add fixed manufacturing overhead cost deferred in inventory (5,000Kg × $4.00) 20,000

———

$150,000

———

PROBLEMS IN VARIABLE AND ABSORPTION COSTING

Problem-1 (Variable costing income statement and reconciliation)


Absorption costing income statement of a company for the first two years is as follows:
Year-1 Year-2

Sales 2,000,000 3,000,000

Less cost of goods sold:

Beginning inventory 0 340,000

Add cost of goods manufactured 1,700,000 1,700,000

————- ————-

Goods available for sale 1,700,000 2,040,000

Less ending inventory 340,000 0

————- ————-

Cost of goods sold 1,360,000 2,040,000

————- ————-

Gross margin 640,000 480,000

Less selling and administrative expenses* 620,000 680,000


————- ————-

Net operating income 20,000 280,000

————- ————-

*6 per unit variable; $500,000 fixed each year.

The manufacturing cost per unit is computed as follows:


Direct materials $16

Direct labor $20

Variable manufacturing overhead $4

Fixed manufacturing overhead $28

——

$68

——

Sales and production for two years:


Year-1 Year-2

Units produced 25,000 25,000

Units sold 20,000 30,000

Required:
1. Prepare a variable costing (contribution margin) income statement.
2. Reconcile net operating income figures.
Solution
(1) Variable costing (contribution margin) income statement:
Year-1 Year-2

Sales 2,000,000 3,000,000

Less variable cost of goods sold:

Beginning inventory 0 200,000

Add variable cost of goods manufactured (25,000 × $40) 1,000,000 1,000,000

————- ————-

Goods available for sale 1,000,000 1,200,000

Less ending inventory (5,000 × $40) 200,000 0


————- ————-

Variable cost of goods sold 800,000 1,200,000

————- ————-

Gross contribution margin 1,200,000 1,800,000

Less variable selling and administrative expenses 120,000 180,000

————- ————-

Contribution margin 1,080,000 1,620,000

————- ————-

Less period costs:

Fixed manufacturing costs 700,000 700,000

Fixed selling and administrative expenses 500,000 500,000

————- ————-

Total period costs 1,200,000 1,200,000

————- ————-

Net operating income/(loss) (120,000) 420,000

————- ————-

(2) Reconciliation:
Year-1 Year-2

Net operating income (loss) under variable costing (120,000) 420,000

Fixed manufacturing cost deferred in inventory ($28 × 5000) 140,000

Fixed manufacturing cost released from inventory ($28 × 5000) (140,000)

———- ———-

Net income under absorption costing 20,000 280,000

———- ———-

For reconciliation of net operating income figures:


When fixed manufacturing overhead cost is deferred in inventory, it is added to the variable costing
income figure and when fixed manufacturing cost is released from inventory, it is deducted from the
variable costing income figure.
Problem-2 (Variable and absorption costing unit product costs and
income statements)
A company manufactures a unique device that is used to boost Wi-Fi signals. The following data relates
to the first month of operation:
Beginning inventory 0

Units produced 40,000

Units sold 35,000

Selling price per unit $120

Selling and administrative expenses:

Variable per unit $4

Fixed (total for the month) $1,120,000

Manufacturing costs:

Direct materials cost per unit $30

Direct labor cost per unit $14

Variable manufacturing overhead cost per unit $4

Fixed manufacturing overhead cost $1,280,000

Management is anxious to see the profitability of newly designed unique booster.


Required:
1. Calculate unit product cost and prepare income statement under variable costing system and absorption
costing system.
2. Prepare income statement under two costing system.
3. Prepare a schedule to reconcile the net operating income under variable and absorption costing system.
Solution:
(1) Calculation of unit product cost:
Variable costing Absorption costing

Direct materials $30 $30

Direct labor $14 $14

Variable manufacturing overhead $4 $4

Fixed manufacturing overhead — $32*

———- ———-
production cost per unit $48 $80

———- ———-

*1,280,000/40,000
(2) Income statements:
Absorption costing:
Sales (35,000 Units × $120) 4,200,000

Less cost of goods sold:

Beginning inventory 0

Add cost of goods manufactured (40,000 Units × $80) 3,200,000

———-

Cost of goods available for sale 3,200,000

Less ending inventory (5,000 Units × $80) 400,000 2,800,000

———- ———-

Gross profit 1,400,000

Less selling and administrative expenses

Fixed 1,120,000

Variable (35,000 Units × $4) 140,000 1,260,000

———- ———-

140,000

———-

Variable costing:
Sales (35,000 Units × $120) 4,200,000

Less variable cost of goods sold:

Beginning inventory 0

Add v. cost of goods manufactured (40,000 Units × $48) 1,920,000

———-
Cost of goods available for sale 1,920,000

Less ending inventory (5,000 Units × $48) 240,000 1,680,000

———- ———-

Gross contribution margin 2,520,000

Less variable selling and administrative expenses 140,000

———-

Contribution margin 2,380,000

Less fixed costs:

Fixed manufacturing overhead 1,280,000

Fixed selling and administrative expenses 1,120,000 2,400,000

———- ———-

Net operating loss (20,000 )

———-

(3) Reconciliation schedule:


Net operating income (loss) under variable costing $(20,000)

Fixed manufacturing overhead cost deferred in inventory (5,000 units × $32) $160,000

———–

Net operating income under absorption costing $140,000

———–

Problem-3 (Impact of change in production on variable and


absorption costing)
AJX company manufactures and sells a single product. Company sold the same number of units this year
as it did last year but generated different profit for two years. The president asks for the explanation of
difference in net operating income for two years.
The income statements of two years are as follows:
Year 1 Year 2

Sales (40,000 units) $2,500,000 $2,500,000

Less cost of goods sold $1,680,000 $1,440,000


————- ————-

Gross margin 820,000 1,060,000

Less selling and administrative expenses 700,000 700,000

————- ————-

Net operating income 120,000 360,000

————- ————-

Sales, production and production for two years are as follows:


Year 1 Year 2

Production in units 40,000 50,000

Sales in units 40,000 40,000

Variable production cost per unit $12 $12

Fixed manufacturing overhead cost $1,200,000 $1,200,000

Variable selling and administrative expenses of AJX are $4.00 per unit sold. A new manufacturing
overhead rate is computed each year.
Required:
1. Calculate unit product cost for both the years under absorption costing and direct costing (variable
costing).
2. Prepare a contribution margin format income statement for two years.
3. Reconcile the net operating income figures for each year under two costing methods.
4. Explain how operations would have different in year 2 if the company had been using just in time (JIT)
manufacturing and inventory control methods.
Solution:
(1) Computation of unit product cost:
Year 1 Year 2

Unit product cost under variable costing $12 $12

Unit product cost under absorption costing $12+$30*=$42 $12+$24**=$36

*1,200,000 / 40,000
**1,200,000 / 50,000

(2) Variable costing income statement:


Year 1 Year 2

Sales 2,500,000 2,500,000


——— ———-

Less variable cost of goods sold:

Beginning inventory 0 0

Variable cost of goods manufactured 480,000 600,000

———- ———-

Cost of goods available for sale 480,000 600,000

Less ending inventory 0 120,000

———- ———-

Cost of goods sold 480,000 480,000

———- ———-

Gross contribution margin 2,020,000 2,020,000

Less variable selling and administrative expenses 160,000 160,000

———- ———-

Contribution margin 1,860,000 1,860,000

———- ———-

Less fixed costs:

Fixed manufacturing overhead expenses 1,200,000 1,200,000

Fixed selling and administrative expenses 540,000 540,000

———- ———-

Total fixed expenses 1,740,000 1,740,000

———- ———-

120,000 120,000

———- ———-

(3) Reconciliation of net operating income:


Year 1 Year 2

Variable costing income statement 120,000 120,000

Fixed manufacturing overhead deferred in inventory 0 240,000


———- ———-

120,000 360,000

———- ———-

(4) Just in time manufacturing method:


If the company had been using just in time manufacturing and inventory control methods in year 2 the
difference in net operating income under variable costing and absorption costing would have been very
little to zero. The central idea of just in time manufacturing is to eliminate inventories. For better
understanding of the impact of JIT read our article variable and absorption costing with just in time (JIT)
manufacturing system.

Problem-4 (Constant production and change in sales – variable and


absorption costing)
Fine Producers Inc. suffered a loss for the first month of operations. Following is the income
statement prepared by the accounting service providers of Fine Producers.
Sales $400,000

Less variable cost of goods sold $160,000

————

Gross contribution margin $240,000

Less variable selling and administrative expenses $60,000

————

Contribution margin $180,000

Less fixed expenses:

Fixed manufacturing overhead $150,000

Fixed selling and administrative expenses $40,000 $190,000

———— ————

Net operating loss $(10,000)

————

The loss created a serious problem because company was planning to use the statement to encourage
investors to purchase the stock of the company. Other relevant data is given below:
Units produced during the first month of operation 50,000
Units sold during the first month of operation 40,000

Variable unit cost:

Direct materials $2.00

Direct labor 1.60

Variable manufacturing overhead expenses 0.40

Variable selling and administrative expenses 1.50

Required:
1. What costing method was used by the accounting service providers to prepare income statement of Fine
Producers Inc? Can an absorption costing income statement show a profit rather than loss? Support your
answer with computations.
2. Prepare company’s income statement using variable costing and absorption costing for the second month
if 60,000 units were sold in the second month and there were no closing inventories.
3. Reconcile the second month’s net operating income under both the costing approaches.
Solution:
(1) Costing method:
Accounting service providers used variable costing method to prepare income statement of Fine
Producers.
Yes, an income statement prepared on the basis of absorption costing can show a profit rather than loss
because a portion of fixed manufacturing overhead cost would be absorbed by ending finished goods
inventory. Under absorption costing, this absorbed fixed cost would be deferred to the next period and
would reduce the burden of current period.
The net operating loss for the first month of operation under variable costing is $10,000 and the cost to be
absorbed under absorption costing system is $30,000*. It results in a net operating profit of $20,000
(30,000 – $10,000). The computations are shown below:
Net profit (loss) under variable costing method $(10,000)

Fixed manufacturing overhead to be deferred in inventory under absorption costing


$30,000*
system.

———

Net operating income if absorption costing is adopted $20,000

———

*10,000 units × $3.00


(2) Income statements for second month:
Absorption costing:
Sales (60,000 Units × $10) $600,000
Less cost of goods sold:

Beginning finished goods inventory 70,000

Direct materials 100,000

Direct labor 80,000

Manufacturing overhead 170,000

———-

Cost of goods manufactured 350,000

———-

Cost of goods available for sale 420,000

Less ending finished goods inventory 0 420,000

———- ———-

Gross profit $180,000

Add selling and administrative expenses:

Fixed 40,000

Variable (60,000 × 1.50 ) 90,000 $130,000

———- ———-

Net operating income $50,000

———-

Variable costing:
Sales (60,000 Units × $10) $600,000

Less cost of goods sold:

Beginning finished goods inventory 40,000

Direct materials 100,000

Direct labor 80,000

Variable manufacturing overhead 20,000

———-

Cost of goods manufactured 200,000


———-

Cost of goods available for sale 240,000

Less ending finished goods inventory 0 240,000

———- ———-

Gross contribution margin 360,000

Less variable selling and admin. expenses 90,000

———-

Contribution margin 270,000

Less fixed costs:

Manufacturing overhead 150,000

Selling and administrative 40,000 190,000

———- ———-

Net operating loss 80,000

———-

(3) Reconciliation:
If the company sells 60,000 units in second month, the sales of the second month will be more than
production. In that case, the fixed manufacturing overhead cost deferred in inventory in first month will be
released from inventory in the second month and the net operating income under absorption costing will
be less than the net operating income under variable costing.
Net operating income under variable costing $80,000

Less fixed manufacturing overhead released from inventory $30,000*

———

Net operating income under absorption costing $50,000

———

*10,000 units × $3.00

Cost Volume and Profit Relationships


Contribution margin
Definition:
Contribution margin is equal to sales revenue less total variable expenses incurred to earn that revenue.
Total variable expenses include both manufacturing and non-manufacturing variable expenses. In
a service firm it is equal to revenue from provision of services less all variable expenses incurred to
provide such services.
Formula:
The equation/formula of contribution margin can be written as follows:
Gross contribution margin = Sales revenue – Variable manufacturing expenses
Contribution margin = Sales revenue – (Variable manufacturing expenses + Variable selling and
administrative expenses)
The concept can best be explained with the help of an example
Example:
Sales revenue = $10,000
Variable expenses:
Manufacturing = $4,000
Marketing and administrative = $1,000
Fixed expenses:
Manufacturing = $2,000
Marketing and administrative = $500
From the above date we can calculate contribution margin and net operating profit as follows:
Contribution margin = $10,000 – ($4,000 + 1,000)
= $5,000
Net operating profit = $5,000 – ($2,000 + $500)
= $2,500
The use of equation to calculate contribution margin figure is just for explaining the concept. For
managerial use, a proper contribution margin income statement is prepared to compute this figure. See
the format of contribution margin income statement.
Importance of Contribution Margin
The amount of contribution margin should be sufficient to cover all fixed costs as well as to contribute
towards profit. If the amount of contribution margin is not enough to cover all fixed costs, the business will
suffer a loss.
Contribution margin figure is even more important for multi product companies. All products are not
equally profitable. High contribution margin products are more profitable because they contribute more for
covering fixed costs and providing for profit. A multi product company can increase its net operating profit
by focusing its attention to increase the sales of high contribution margin products. Read this article to
understand the importance of higher contribution margin products for a multi product company.

Contribution margin ratio


Posted in: Cost Volume and Profit Relationships

Share8
Definition:
Contribution margin ratio (CM ratio) is the ratio of contribution margin to net sales.
Formula:
Contribution margin ratio is calculated by dividing contribution margin figure by the net sales figure. The
formula can be written as follows:

The ratio can also be shown in percentage form as follows:

The numerator of the formula i.e., contribution margin can be calculated using simple contribution margin
equation or by preparing a contribution margin income statement.
Examples:
Example 1
The total sales revenue of Black Stone Crushing Company was $150,000 for the last year. The fixed and
variable expenses data of the last year is given below:
Variable expenses:

Manufacturing $60,000

Marketing and administrative 30,000

Fixed:

Manufacturing 10,000

Marketing and administrative 8,000

Required: Calculate contribution margin ratio and also express it in percentage form.
Solution:
Contribution margin = $150,000 – ($60,000 + $30,000)
$150,000 – $90,000
= $60,000
Contribution margin ratio = $60,000/$150,000
= 0.4
Contribution margin percentage = ($60,000/$150,000) × 100
= 40%
The contribution margin is 40% of net sales.
Example 2:
The Monster company manufactures and sells two products. The data relating to sales and expenses for
the last six months is given below:
Product A Product B
Sales in units 3,200 2,000

Sale price per unit $12 $10

Variable expenses to manufacture a unit of product $5 $4

Variable expenses to sell a unit of product $2 $1

The total fixed expenses for the last six months were $60,000. Out of these fixed expenses, 50% were
manufacturing and remaining 50% were related to marketing and administrative activities. There were no
beginning and ending inventories.
Required: Which product has the higher contribution margin ratio?
Solution:
Product A Product B

Sales 38,400 20,000

Less variable cost of goods sold 16,000 8,000

Gross contribution margin 22,400 12,000

Less variable marketing and administrative expenses 6,400 2,000

Contribution margin 16,000 10,000

Contribution margin ratio (contribution margin/sales) 0.42 0.50

Product A’s contribution margin ratio is 0.42 or 42% where as product B’s contribution margin ratio is 0.5
or 50%. Product B’s contribution margin ratio is higher than product B.

Contribution margin income statement


Posted in: Cost Volume and Profit Relationships

Share23

The basic difference between a traditional income statement and a contribution margin income
statement lies in the treatment of variable and fixed expenses. This difference of treatment of two types
of costs affects the format and uses of two statements.
In this article, we shall discuss two main differences of two income statements- the difference of format
and the difference of usage.
(1).The difference of format:
In a traditional income statement, cost of goods sold (variable + fixed) is subtracted from sales revenue to
obtain gross profit figure and marketing and administrative expenses (variable + fixed) are then
subtracted from gross profit figure to obtain net operating income.
In a contribution margin income statement, variable cost of goods sold is subtracted from sales revenue
to obtain gross contribution margin. The variable marketing and administrative expenses are then
subtracted from gross contribution margin to obtain contribution margin. From contribution margin figure
all fixed expenses are subtracted to obtain net operating income. The following simple formats of two
income statements can better explain this difference.

Notice that a traditional income statement calculates gross profit and net profit whereas a
contribution margin income statement calculates gross contribution margin, contribution
margin and net profit.
(2). The difference of use:
A traditional income statement is prepared under a traditional absorption costing (full costing) system and
is used by both external parties and internal management. As this statement fulfills the requirements of
external parties to great extent, companies are required to follow applicable accounting standards such
as generally accepted accounting principles (GAAP) or international accounting standards (IAS).
A contribution margin income statement, on the other hand, is a purely management oriented format of
presenting revenues and expenses that helps in various revenues and expense related decision making
processes. For example, a multi-product company can measure profitability of each product by preparing
a product viz contribution margin income statement and decide which product to continue and which one
to drop. Companies are not required to present such statements to any external party, so there is no need
to follow GAAP or IAS.

Examples:
The following examples explain the difference between traditional income statement and variable costing
income statement.
Example 1 – single product:
The Friends company is a single product company. The following data is available for the month of March
2014.
Sale price per unit $30

Number of units manufactured and sold during the month 2,000

Variable manufacturing cost per unit $12

Variable selling and administrative cost per unit $4

Fixed manufacturing cost per month $6,000

Fixed marketing and administrative cost per month $2,000

Company does not maintain finished goods and work in process inventory.
Required: Prepare a traditional income statement and a contribution margin income statement.
Solution:
Traditional format:
Sales (2,000 units × $30) 60,000

Less cost of goods sold (2,000 units × $15*) 30,000

Gross profit 30,000

Less marketing and administrative expenses

Variable (2,000 units × $4) $8,000

Fixed $2,000 10,000


Net operating income 20,000

*$12 + (6,000/2,000)

Contribution margin format:


Sales (2,000 units × $30) 60,000

Less variable cost of goods sold (2,000 units × $12) 24,000

Gross contribution margin 36,000

Less variable marketing and administrative expenses (2,000 units × $4) 8,000

Contribution margin 28,000

Less fixed expenses/period expenses

Manufacturing 6,000

Marketing and administrative 2,000 8,000

Net operating income 20,000

Example 2 – multi product company:

HC company is a US based multi product company. The company is currently manufacturing and selling
four products successfully. The data for the year 2013 is given below:
Product A Product B Product C Product D Total

Number of units manufactured and sold 2,000 1,500 1,000 800

Sales price per unit $50 $75 $60 $90 $275

Variable manufacturing expenses per unit $30 $45 $25 $50 $150

Variable marketing and administrative expenses $5 $9 $10 $7 $30

Total fixed manufacturing expenses $37000

Total fixed marketing and administrative expenses $25000

The marketing department with the cooperation of research and development department has proposed
the production of a new product. Because of limited resources, the new product can only be
manufactured if one of the existing products is dropped. The new product is very attractive and is
expected to generate a contribution margin of 35% but after a careful review of market share, competition
in the market, available resources, and target profit of the next year, the company will only stop
manufacturing a product if its contribution margin is less than 30% of its ultimate sales price.
Required:
1. Prepare a product viz contribution margin income statement of HC company.
2. Will HC company start manufacturing proposed product? Use the answer of requirement 1 for your
calculations.
Solution:
(1) Contribution margin income statement:
Product A Product B Product C Product D Total

Sales $100,000 $112,500 $60,000 $72,000 $344,500

Variable cost of goods sold 60,000 67,500 25,000 40,000 192,500

Gross contribution margin 40,000 45,000 35,000 32,000 152,000

Variable marketing and


10,000 13,500 10,000 5,600 39,100
administrative expenses

Contribution margin 30,000 31,500 25,000 26,400 112,900

Total fixed manufacturing


37,000
expenses

Total fixed marketing and


25,000
administrative expenses

Total fixed expenses 50,900

Net operating income 52,400

(2). Introduction of new product:


The production of newly proposed product is possible only if at least one of the existing products is
dropped. The company will manufacture newly proposed product because product B does not generate a
contribution margin of 30% or above. The calculations are given below:
Product A: (30,000/100,000) × 100 = 30%
Product B: (31,500/112,500) × 100 = 28%
Product C: (25,000/60,000) × 100 = 41.67%
Product D: (26,400/72,000) × 100 = 36.67%
The product B will be replaced by the newly proposed product.
Break-even point analysis
Explanation of break-even point:
The point at which total of fixed and variable costs of a business becomes equal to its total revenue is
known as break-even point (BEP). At this point, a business neither earns any profit nor suffers any loss.
Break-even point is therefore also known as no-profit, no-loss point or zero profit point. Calculation of
break-even point is important for every business because it tells business owners and managers how
much sales are needed to cover all fixed as well as variable expenses of the business or the sales
volume after which the business will start generating profit. The computation of sales volume required to
break-even is known as break-even analysis. The concept explained above can also be presented as
follows:

After reading this article you will be able to compute the break-even point of a single
product company using two popular methods – equation method and contribution margin method. First
we shall compute break-even point using these two methods and then present the information graphically
(preparation of break-even chart).
Computation of break-even point:
(1). Use of equation method:
The application of equation method facilitates the computation of break-even point both in units and in
dollars. As we have already described that the sales are equal to total variable and fixed expenses at
break-even point, the equation can therefore be written as follows:
Sp × Q = Ve × Q + Fe
Or
SpQ = VeQ + Fe
Where;
Sp = Sales price per unit.
Q = Number (quantity) of units to be manufactured and sold during the period.
Ve = Variable expenses to manufacture and sell a single unit of product.
Fe = Total fixed expenses for the period.
Notice that the left hand side of the equation represents the total sales in dollars and the right hand side
of the equation represents the total cost. If the information about sales price per unit, variable expenses
per unit and the total fixed expenses is available, we can solve the equation for ‘Q’ to find the number of
units to break-even. The break-even point in units can then be multiplied by the sales price per unit to
calculate the break-even point in dollars. Suppose, for example, you run a manufacturing business that is
involved in manufacturing and selling a single product. The annual fixed expenses to run the business are
$15,000 and variable expenses are $7.50 per unit. The sale price of your product is $15 per unit. The
number of units to be sold to break even can be easily calculated using equation method:
Sp × Q = Ve × Q + Fe
15 × Q = 7.5 × Q + 15,000
15 Q = 7.5 Q + 15,000
15Q – 7.5Q = 15,000
7.5Q = 15,000
Q = 15,000 / 7.5
Q = 2,000 units
The break-even point in units is 2,000 units and the break-even point in dollars can be computed as
follows:
= (2,000 units) × ($15)
= $30,000
(2). Use of contribution margin method:
The method described above is known as equation method of calculating break-even point. Some
people use another method called contribution margin method (read about contribution margin and its
calculation). Under this method, the total fixed expenses are divided by contribution margin per unit.
Consider the following computations:
Total fixed expenses / Contribution margin per unit
= 15,000 / 7.5*
= 2,000 units
or
= (2,000 units) × ($15)
= $30,000
*$15 – $7.5
A little variation of this method is to divide the total fixed expenses by the contribution margin ratio (CM
ratio). Doing so results in break-even point in dollars. It is shown below:
Total fixed expenses / Contribution margin ratio
= $15,000 / 0.5*
= $30,000
*($15 – $7.5)/$15
Graphical presentation (Preparation of break-even chart or CVP
graph):
The graphical presentation of dollar and unit sales needed to break-even is known as break-even
chart or CVP graph:
Explanation of the graph:
1. The number of units have been presented on the X-axis (horizontally) where as dollars have been
presented on Y-axis (vertically).
2. The straight line in red color represents the total annual fixed expenses of $15,000.
3. The blue line represents the total expenses. Notice that the line has a positive or upward slop that
indicates the effect of increasing variable expenses with the increase in production.
4. The green line with positive or upward slop indicates that every unit sold increases the total sales
revenue.
5. The total revenue line and the total expenses line cross each other. The point at which they cross each
other is the break-even point. Notice that the total expenses line is above the total revenue line before the
point of intersection and below after the point of intersection. It tells us that the business suffers a loss
before the point of intersection and makes a profit after this point. The break-even point in the above
graph is 2,000 units or $30,000 that agrees with the break-even point computed using equation and
contribution margin methods above.
6. The difference between the total expenses line and the total revenue line before the point of intersection
(BE point) is the loss area. The loss area has been filled with pink color. Notice that this area reduces as
the number of units sold increases. It means every additional unit sold before the break-even point
reduces the loss.
7. The difference between the total expenses line and the total revenue line after the point of intersection
(BE point) is the profit area. The profit area has been filled with green color. Notice that this area
increases as the number of units sold increases. It means every additional unit sold after the break-even
point increases the profit of the business.
Break-even analysis with multiple products
The method of calculating break-even point of a single product company has been discussed in
the break-even point analysis article. In this article, I would explain the procedure of calculating break-
even point of a multi product company. A multi-product company means a company that sells two or more
products.
The procedure of computing break-even point of a multi product company is a little more complicated
than that of a single product company.
Formula:
A multi product company can compute its break-even point using the following formula:

For computing break-even point of a company with two or more products, we must know the sales
percentage of individual products in the total sales mix. This information is used in computing weighted
average selling price and weighted average variable expenses.
In the above formula, the weighted average selling price is worked out as follows:
(Sale price of product A × Sales percentage of product A) + (Sale price of product B × Sale percentage of
product B) + (Sale price of product C × Sales percentage of product C) + …….
and the weighted average variable expenses are worked out as follows:
(Variable expenses of product A × Sales percentage of product A) + (Variable expenses of product B ×
Variable expenses of product B) + (Variable expenses of product C × Sales percentage of product C) +
…….
When weighted average variable expenses per unit are subtracted from the weighted average selling
price per unit, we get weighted average contribution margin per unit. Therefore, the above formula can
also be written as follows:

An example would be very helpful to understand the whole procedure. Consider the following example of
a multi product company:
Example:
The Monster company manufactures three products – product X, product Y and product Z. The
variable expenses and sales prices of all the products are given below:
Product X Product Y Product Z

Sales price per unit $200 $100 $50

Variable cost per unit $100 $75 $25

The total fixed expenses of the company are $50,000 per month. For the coming moth. Monster expects
the sale of three products in the following ratio:
Product X: 20%;
Product Y: 30%;
Product Z: 50%
Required: Compute the break-even point of Monster company in units and dollars for the coming month.
Solution:
Monster company sells three products and is, therefore, a multi product company. Its break-even point
can be computed by applying the above formula:

= $50,000 / $95* – $55**


= $50,000 / $40
= 1,250 units
*Weighted average selling price:
= ($200 × 20%) + ($100 × 30%) + ($50 × 50%)
= $40 + $30 + $25
= $95
**Weighted average variable expenses:
= ($100 × 20%) + ($75 × 30%) + ($25 × 50%)
= $20 + 22.50 + 12.50
= $55
The company will have to sell 1,250 units to break-even. Now I would compute the number of units of
each product to be sold:
Products No. of units

Product X (1,250 × 20%) 250

Product Y (1,250 × 30%) 375

Product Z (1,250 × 50%) 625

———

Total 1,250

———

As the number of units of each individual product to be sold have been computed, I can compute
the break even point in dollars as follows:
Product X (250 units × $200) $50,000

Product Y (375 units × $100) $37,500

Product Z (625 units × $50) $31,250

————
Break-even point in dollars $118,750

————

The break-even point of Monster company is $118,750. It can be verified by preparing a contribution
margin income statement as follows:
Sales (break-even point in dollars) $118,750

Less variable expenses $68,750*

————

Contribution margin 50,000

Less fixed expenses 50,000

————

Net operating income 0

————

*(250 units × $100) + (375 units × $75) + (625 units × $25) = $68,750
Sales mix and break-even point analysis
Explanation of the sales mix:
The proportion in which a multi-product company sells its products is referred to as sales mix.
Companies involved in selling two or more products try to sell their products in a proportion or mix that
maximizes their total profit.
A business with low sales volume may earn more profit than a business with high sales volume if it has a
large proportion of high margin products in its sales mix.
A business is not always free to sell any number of units of a product that generates the highest margin
for the company. Because the sale depends on a number of external factors such as user’s demand for
the product, supply of raw materials, company’s production capacity, restrictions imposed by government
etc. Since sale depends on some uncontrollable factors, the ultimate purpose of the companies is to find
a sales mix that will generate the highest profit for them.
Shift in sales mix and break-even point:
Usually, different products have different sales prices, variable expenses and contribution margin.
Therefore, any change in proportion in which the products are sold has significant impact on the break-
even point. This change is known as ‘change in sales mix’ or ‘shift in sales mix’. For better explanation,
consider the following example:
Example:
The NORAN company sells two products; product X and product Y. The information about sales price,
variable expenses per unit and total fixed expenses is given below:
Product X Product Y
Sales price $50 per unit $100 per unit

Variable expenses $30 per unit $40 per unit

The total monthly fixed expenses of the company are $270,000. The company wants to generate a sales
revenue of $1,000,000 in the next month. To obtain this goal the company has the following options:
(i). Sell 6,000 units of product X and 7,000 units of product Y.
(ii). Sell 14,000 units of product X and 3,000 units of product Y.
Required:
1. Prepare contribution margin income statement and calculate break-even point if NORAN decides to
select option (i).
2. Prepare contribution margin income statement and calculate break-even point if NORAN decides to
select option (ii).
3. Whichever is the better option, (i) or (ii)?
4. Explain the reason of change in break-even point in dollars (if any).
Solution:
(1). If option (i) is selected:
NORAN Company
Contribution margin income Statement
For the month of ………….
Product X Product Y Total

Sales $300,000 $700,000 $1,000,000

Less variable expenses $180,000 $280,000 $460,000

————- ————- ————-

Contribution margin $120,000 420,000 $540,000

————- ————-

Less fixed expenses $270,000

————-

Net operating income $270,000

————-

Break-even point = Total fixed expenses / Overall contribution margin ratio


= $270,000/.54*
= $500,000
*540,000/1,000,000
(2). If option (ii) is selected:
NORAN Company
Contribution margin income Statement
For the month of ………….
Product X Product Y Total

Sales $700,000 $300,000 $1,000,000

Less variable expenses $420,000 $120,000 $540,000

————- ————- ————-

Contribution margin $280,000 180,000 $460,000

————- ————-

Less fixed expenses $270,000

————-

Net operating income $190,000

————-

Break-even point = Total fixed expenses / Overall contribution margin ratio


= $270,000/.46*
= $586,957
*300,000/1,000,000
(3) The better option:
Option (i) is better than option (ii) because it generates more net operating income.
(4). The reason of change in break-even point:
A change in sales mix usually have a strong effect on the break-even point. The break-even point has
increased from $500,000 to $586,957 because the shift in sales mix from high margin product (product Y)
to low margin product (product X) has dropped the overall contribution margin ratio from 0.54 to 0.46.
A shift in sales mix from high contribution margin product to low contribution margin product increases the
dollar sales required to break-even while a shift from low contribution margin product to high contribution
margin product reduces the dollar sales required to break-even.
Margin of safety
Posted in: Cost Volume and Profit Relationships

Share20

Margin of safety (MOS) is the difference between actual sales and break even sales. In other words, all
sales revenue above the break-even point represents the margin of safety. For example, if actual sales
for the month of December 2015 are $2,500,000 and the break-even sales are $1,500,000, the difference
of $1,000,000 is margin of safety.
Margin of safety is an important figure for any business because it tells management how much reduction
in revenue will result in break-even. A higher MOS reduces the risk of business losses. Generally, the
higher the margin of safety, the better it is.
Formula:
The formula or equation of MOS is given below:
MOS = Actual or budgeted sales – Sales required to break-even
Margin of safety is also expressed in the form of ratio or percentage that is calculated by using the
following formulas:
MOS ratio = MOS/Actual or budgeted sales
MOS percentage = (MOS/Actual or budgeted sales) × 100
Example 1:
The following data relates to Noor enterprises for the Month of June 2015.
Sales (3,500 units @ $20/unit) $70,000

Contribution margin per unit $12

Total fixed expenses for the month $15,000

There was no opening and closing finished goods inventory in stock.


Required: Calculate margin of safety for the Noor enterprises using above data.
Margin of safety = Actual sales – Break-even sales
=$70,000 – $25,000*
=$45,000
or
45,000/70,000
= 0.6429
or
64.29%
The margin of safety of Noor enterprises is $45,000 for the moth of June. It means if $45,000 in sales
revenue is lost, the profit will be zero and every dollar lost in addition to $45,000 will contribute to loss.
*Break-even sales
$15,000/$12 = 1,250 units
1250 units × $20 = $25000
Example 2:
The break even point of Best Inc. is $65,000 for the first quarter of the year 2016. If margin of safety is
$45,000, calculate the actual sales for the first quarter.
Solution
Margin of safety = Actual sales – Break-even sales
$45,000 = Actual sales – $65,000
Actual sales = $45,000 + $45,000
Actual sales = $90,000
Target Profit Analysis
Sometime management wants to earn a certain amount of profit during a certain period of time. This
certain amount of profit is commonly known as target profit.
Target profit analysis is about finding out the estimated business activities to perform to earn a
target profit during a certain period of time. Among these activities, management is especially interested
to find out the sales volume required to generate a target profit.
In break-even point analysis article, we used equation method and contribution margin method
to calculate break-even point of a company. The same formulas, with a little modification, can be used to
calculate the sales both in units and in dollars to earn a target profit during a certain period of time.
Equation method for target profit:
The target profit equation is given below:
Sp × Q = Ve × Q + Fe + Tp
or
SpQ = VeQ + Fe + Tp
Where;
Sp = Sales price per unit.
Q = Number (quantity) of units to be manufactured and sold during the period.
Ve = Variable expenses to manufacture and sell a single unit of product.
Fe = Total fixed expenses for the period.
Tp = Target profit for the period.
Contribution margin method for target profit:
Under this method, the target profit is added in the total fixed expenses and the resultant figure is then
divided by the unit contribution margin.

Notice that to get target profit formulas or equations, we have just included the target profit to break-even
point formulas.
Examples:
The HK company manufactures a single product – product X. A unit of product X is sold to customers for
$80. The per unit variable expense and the total expected fixed expenses for the first quarter of the year
2012 are as follows:
 Variable expenses to manufacture and sell a unit of product X: $50
 Total fixed expenses for the first quarter of the year 2012: $40,000
The company wants to earn a profit of $80,000 for the first quarter of the year 2012.
Required:
Calculate sales in units and in dollars to earn a target profit of $80,000 during the first quarter of 2012
using:
1. Equation method
2. Contribution margin method.
Solution:
(1). Equation method:
SpQ = VeQ + Fe + Tp
$80Q = $50Q + $40,000 + $80,000
$80Q – $50Q = $40,000 + $80,000
$30Q = $120,000
Q = $120,000 / $30
Q = 4,000
The company will need to sell 4,000 units of product X to earn a profit of $80,000. We can calculate the
sales in dollars by simply multiplying the number of units to be sold by the sales price per unit as follows:
= 4,000 units × $80
= $320,000
Contribution margin method:
Target profit + fixed expenses)/contribution margin per unit
($40,000 + $80,000) / $30*
4,000 units
Or
4,000 units × $80
$320,000
*Unit contribution margin is equal to sales price per unit less variable expenses per unit i.e., $80 – $50.

EXERCISES
Exercise-1 (Target profit analysis, break-even point)
PNG electric company manufactures a number of electric products. Rechargeable light is one of the
PNG’s products that sells for $180/unit. Total fixed expenses related to rechargeable electric light are
$270,000 per month and variable expenses involved in manufacturing this product are $126 per unit.
Monthly sales are 8,000 rechargeable lights.
Required:
1. Compute break-even point of the company in dollars and units.
2. According to a research conducted by sales department, a 10% reduction in sales price will result in 25%
increase in unit sale. Prepare two income statements in contribution margin format, one using the current
price and one using proposed price (10% below the old sales price).
3. Compute the number of rechargeable lights to be sold to earn a net operating income of $189,000 per
month (use original data).
Solution:
(1) Computation of break-even point:
(a). Break even point in units:
Fixed expenses / Contribution margin per unit
270,000 / 54*
= 5,000 units
*$180 – $126
(b). Break-even point in dollars can be computed by multiplying break-even point in units by sales price
per unit as shown below:
5,000 units × $180
=$900,000
(2) Income statements:
(a) Income statement under current operations:
Total Per unit

Sales (8,000 lights) 1,440,000 $180

Less variable expenses 1,008,000 $126

———— ————

Contribution margin 432,000 $54

Less fixed expenses 270,000 ————

————

Net operating income 162,000

————

(b) Income statement under proposed operations:


Total Per unit

Sales (10,000 lights) 1,620,000 $162

Less variable expenses 1,260,000 $126

———— ————

Contribution margin 360,000 $36

Less fixed expenses 270,000 ————

————

Net operating income 90,000

————

The proposal should not be accepted because it will reduce the contribution margin from $54 per unit to
$36 per unit and net operating income from $162,000 to $90,000.
(3) Target profit analysis:
We can compute the target income using following equation
Sales = Variable expenses + Fixed expenses + Profit
$180Q = $126Q + 270,000 + $189,000
$180Q – $126Q = $459,000
$54Q = $459,000
Q = $459,000 / $54
Q = 8,500 Units
On the basis of original data, company needs to sell 8,500 rechargeable lights to earn a profit of
$189,000.
Exercise-2 (Break-even analysis of a multiproduct company)
PQR company sells two products. The total fixed expenses of the company are 1,197,000. The monthly
data of PQR is as follows:
Products

Product A Product B Total

Sales $1,400,000 $600,000 $2,000,000

Contribution margin ratio 60% 70% ?

Required:
1. Prepare contribution margin income statement for the company.
2. Calculate break-even point in dollars.
Solution:
(1) Income statement:
Product A Product B Total

Amount Percent Amount Percent Amount Percent

Sales 1,400,000 100% 600,000 100% 2,000,000 100%

Less variable expenses 560,000 40% 180,000 30% 740,000 37%

———- —– ———- —– ———- —–

840,000 60% 420,000 70% 1,260,000 63%

Less fixed expenses ———- —– ———- —– 1,197,000 —–

———-

63,000

———-

(2) Computation of break-even point:


The PQR company sells two products. Its break-even point can be easily computed by dividing the total
fixed expenses by overall contribution margin ratio (CM ratio).
Fixed expenses / Overall CM ratio
1,197,000 / .63
$1,900,000
Exercise-3 (Change in sales volume, sales price, variable and fixed
costs)
Aladin company manufactures small battery that is used in clocks, toys and some other electronic
devices. The last month’s income statement of Aladin is given below:
Total Per unit

Sales (30,000 batteries) $300,000 $10

Less variable expenses $180,000 $6

———- —-

Contribution margin $120,000 $4

Fixed expenses $100,000 —-

———-

Net operating income $20,000

———-

Required:
Prepare Aladin’s new income statement under each of the following conditions:
1. The sales volume increases by 15%.
2. The selling price decreases by 20% per unit, and the sales volume increases by 30%.
3. The selling price increases by 50% per unit, fixed expenses increase by $20,000 and the sales volume
decreases by 5%.
4. Variable expenses increase by 20% per unit, the selling price increases by 12%, and the sales volume
decreases by 10%.
Solution:
(1). Sales volume (number of units sold) increases by 15%:
Total Per unit

Sales (34,500) $345,000 $10

Less variable expenses $207,000 $6

———- —-

Contribution margin $138,000 $4

Fixed expenses $100,000 —-

———-

Net operating income $38,000


———-

(2). Selling price decreases by 10% and the sales volume increases by 30%:
Total Per unit

Sales (39,000 Units) $312,000 $8

Less variable expenses $234,000 $6

———- —-

Contribution margin $78,000 $(2)

Fixed expenses $100,000 —-

———-

Net operating loss $(22,000)

———-

(3). Selling price increases by 50%, fixed expenses increase by $20,000 and the
sales volume decreases by 5%:
Total Per unit

Sales (28,500 Units) $427,500 $15

Less variable expenses $171,000 $6

———- —-

Contribution margin $256,500 $9

Fixed expenses $120,000 —-

———-

Net operating income $136,500

———-

(4). Variable expenses increase by 20% per unit, the selling price increases by 12%,
and the sales volume decreases by 10%.
Total Per unit

Sales (27,000 Units) $302,400 $11.20

Less variable expenses $194,400 $7.20

———- —-
Contribution margin $108,000 $4.00

Fixed expenses $100,000 —-

———-

Net operating income $8,000

———-

Exercise-4 (Degree of operating leverage, contribution margin


income statement)
TLK Ltd. manufactures small size fans to be used in load shedding areas. Each fan has a rechargeable
battery and a built in charging circuit. TLK sells a fan for $120. The annual sale is 30,000 fans. Variable
and fixed cost data is given below:
Variable expenses $84 per fan

Fixed expenses $900,000 per year

Required:
1. Prepare contribution margin income statement and compute the degree of operating leverage.
2. Next year the sales are expected to increase by 7,500 fans. Compute (a) the expected percentage
increase in net operating income (b) expected increase in net operating income and (c) expected total net
operating income for the next year. (You don’t need to prepare an income statement, use the degree of
operating leverage for your answers).
Solution:
(1) Income statement:
Total Per unit

Sales (30,000 fans) $3,600,000 $120

Less variable expenses (30,000 fans) $2,520,000 $84

————– ——-

Contribution margin $1,080,000 $36

——-

Less fixed expenses $900,000

————–

$180,000

————–
Degree of operating leverage = Contribution margin / Net operating income
= 1,080,000 / 180,000
=6
(2) Expected increase in net operating income:
The degree of operating leverage (computed above) is 6. The net operating income is expected to
increase 6 times as fast as the sales of TLK company.
(a). Expected percentage increase in net operating income:
(Percentage change in sales × 6)
=*25% × 6
=150%
*7,500 / 30,000
(b). Expected increase in total net operating income:
= $180,000 × (150/100)
= $270,000
(c). Expected total net operating income:
= 180,000 + 270,000
= $450,000
Exercise-5 (CM ratio, break-even analysis, target profit analysis,
margin of safety)
Following is the contribution margin income statement of a single product company:
Total Per unit

Sales $1,200,000 $80

Less variable expenses $840,000 $56

———– ——-

Contribution margin 360,000 $24

Less fixed expenses 300,000 ——-

———–

Net operating income $60,000

———–

Required:
1. Calculate break-even point in units and dollars.
2. What is the contribution margin at break-even point?
3. Compute the number of units to be sold to earn a profit of $36,000.
4. Compute the margin of safety using original data.
5. Compute CM ratio. Compute the expected increase in monthly net operating if sales increase by
$160,000 and fixed expenses do not change.
Solution:
(1) Break-even point in units and dollars:
Fixed expenses / Unit contribution margin
$300,000 / $24
12500 units
or
(12,500 units × $80) = $1000000
(2) Contribution margin at break-even point:
Contribution margin must be $300,000 at break-even point because it will cover fixed costs and nothing
will remain to go towards profit.
(3) Computation of target profit (contribution margin method):
(Fixed expenses + Target profit) / Unit contribution margin
($300,000 + $36,000) /$24
Company must sell 14,000 units of product to earn a target profit of $36,000.
(4) Margin of safety in dollars and percentage:
Actual or budgeted sales – sales required to break-even
$1,200,000 – $1,000,000
$200,000
or
$200,000 / 1,200,000 = 16.67%
(5) CM ratio and expected change in net operating income:
Contribution margin / Total sales
360,000 / 1,200,000
30%
If the sales are increased by $160,000 without any change in fixed expenses, the net operating income
will be increased by:
$160,000 × CM ratio of 30%
$48,000
PROBLEMS
Problem-1 (Cost structure, target profit analysis, CM ratio, break-
even analysis)
Zoltrixound company manufactures high quality speakers for desktop and laptop computers. Last month
Zoltrixound suffered a loss of $18,000. The income statement of the last month is as follows:
Sales (13,500 units × $40) 540,000

Less variable expenses 378,000

———

Contribution margin 162,000

Less fixed expenses 180,000

———

Net operating loss $(18,000)

———

Required:
1. Compute the break-even point and contribution margin ratio of Zoltrixound company?
2. Sales department feels that if monthly advertising budget is increased by $16000, the sales will be
increased by $140,000. Show the effect of this change.
3. If sales price is reduced by 20% and monthly advertising expenses are increased by $70,000, the unit
sales are expected to increase by 100%. Show the effect of this change by preparing a new income
statement of Zoltrixound company.
4. The Zoltrixound wants to make the packing of its product more attractive. The new packing would
increase cost by $1.20 per unit. Assuming no other changes, compute the number of units to be sold to
earn a net operating income of $9,000.
5. The company is planning to purchase a new machine. The installation of new machine will increase fixed
cost by $236,000 and decrease unit variable expenses by 50%.
(a). Compute the CM ratio and break-even point if the new machine is installed.
(b). Company expects a sale of 20,000 units for the next month. Prepare two income statement, one
assuming that the machine is not installed and one assuming that it is installed.
(c) Should the company install new machine. Give your recommendations.
Solution:
(1) Computation of CM ratio and break-even point:
CM ratio = $162,000 / $540,000
=0.3 or 30%
Break-even point = $180,000 / $12
= 15,000 units or $600,000
(2) Effect of increase in advertising budget by $16,000 and sales by $140,000:
Increase in sales revenue $140,000

Less increase in expenses:

Variable (3,500* units × $28**) 98,000

Fixed (monthly advertising budget) $16,000 $114,000

——– ———-

$26,000

———-

If sales are increased by $140,000, the company will earn a profit of $8,000 ($26,000 – $18,000) rather
than suffering a loss.
*$140,000 /$40 = 3,500 units
**$378,000 /$13,500 = $28
(3) Effect of reduction in sales price by 20%, increase in monthly advertising budget by $70,000
and increase in unit sales by 100%:
Sales (27,000 units × $32 ) $864,000

Less variable expenses (27,000 units × $28 ) $756,000

———-

Contribution margin $108,000

Less fixed expenses $250,000

———-

Net operating loss $(142,000)

———-

(4) Computation of the number of units to be sold to earn $9,000:


($180,000 + $9,000) / $10.8*
17,500 Units
*The variable expenses in original income statement are $28. The new packing will increase variable
expenses and reduce contribution margin per unit by $1.20 as shown below:
[$40 – ($28 + $1.20)] = $10.8
(5) Installation of new machine:
(a).
Computation of new CM ratio:
351,000 / 540,000
0.65%
Break-even point:
416,000 / $26
16,000 units
or
16,000 units × $40 = $640,000
(b).
If new machine is not installed:
Sales (20,000 units × $40) 800,000

Less variable expenses (20,000 units × $28) 560,000

———–

Contribution margin 240,000

Less fixed expenses 180,000

———–

Net operating income 60,000

———–

If new machine is installed:


Sales (20,000 units × $40) 800,000

Less variable expenses (20,000 units × $14) 280,000

———–

Contribution margin 520,000

Less fixed expenses ($180,000 + $236,000) 416,000

———–

Net operating income 104,000

———–

(c). If only the monthly net operating income is considered, Zoltrixound should purchase and install the
new machine because it will generate more net operating income . Decrease in variable expenses are
more than the increase in fixed expenses if 20,000 units are produced.
Decrease in variable expenses: (20,000 units × $14) = 280,000
Increase in fixed expenses: $236,000
Problem-2 (Basic CVP analysis, CVP graph or break even chart,
break-even analysis)
Beta company sells blouses in Washington, USA. Blouses are imported from Pakistan and are sold to
customers in Washington at a profit. Salespersons are paid basic salary plus a decent commission
on sales made by them. Sales and expense data is given below:
Selling price per blouse $80.00

———

Variable expenses per blouse:

Invoice cost $36.00

Sales commission $14.00

———

Total $50.00

———

Annual fixed expenses:

Rent $160,000

Marketing $300,000

Salaries $140,000

———

Total $600,000

———

Required:
1. Compute the number of units to be sold to break-even.
2. Prepare a CVP graph (break-even chart) and show the break-even point on the graph.
3. If the manage is paid a commission of $6 blouse (in addition to the salesperson’s commission), what will
be the effect on company’s break-even point?
4. As an alternative to (3) above, company is thinking to pay $6 commission to manager on each blouse
sold in excess of break-even point. What will be the effect of these changes on the net operating income
or loss of the Beta company if 23,500 blouses are sold in a year?
5. Refer to the original data. What will be the break-even point of the company if commission is entirely
eliminated and salaries are increased by $214,000? Should the company make this change?
Solution:
(1) Calculation of break-even point:
Fixed expenses / Contribution margin per unit
$600,000 / $30
20,000 units
or
20,000 units × $80 = $1,600,000
(2) CVP graph or break-even chart:

(3) Break-even point if manager is also paid a commission of $6 per blouse sold:
The payment of a commission of $6 to manager will decrease the unit contribution margin and increase
the number of units required to sell to break-even.
$600,000 / $24
25,000 Units
Now the company requires 25000 units or $2,000,000 in sales just to break-even.
(4) Effect on net operating income or loss if manager is paid a commission of $6 on each blouse
sold after break-even point:
Sales (23,500 × $80) $ 1,880,000

Less variable expenses (23,500 × $50) 1,175,000

————

705,000

Less manager’s commission [(23,500 – 20,000) × 6] 21,000

————

684,000

Fixed expenses 600,000


————

Net operating income 84,000

————

(5) Break-even point after elimination of commission and increase in salaries:


$814,000 /$44
18,500 units
or
18,500 × $80 = $14,80,000
Fixed cost after change: $600,000 + 214,000 = 814,000
Unit contribution margin after change: $80 – $36 = $44
With the new system, Beta company will start making profits after selling 18,500 units but with the old
system company needs to sell 20,000 units before making any profit. The change should, therefore, be
implemented.
Problem-3 (Shift in sales mix, break-even analysis of a multiproduct
company)
The Modern Digital World company sells three products – wireless keyboards, wireless mouses and
wireless headphones. All the products are compatible with general laptop and desktop computers.
The budgeted contribution margin income statement of the Modern Digital World for a month is given
below:
Wireless Wireless Wireless
Total
keyboards mouses headphones

Sales $480,000 100% $200,000 100% $320,000 100% $1,000,000 100%

Less variable expenses 144,000 30% 160,000 80% 176,000 55% 480,000 48%

——– —– ——– —– ——– —– ——– —–

Contribution margin $336,000 70% $40,000 20% $144,000 %45 520,000 52%

——– —– ——– —– ——– —– —–

Less fixed expenses 447,200

——–

Net operating income 72,800

——–

Budgeted break-even point:


Fixed expenses / CM ratio = 447,200 / 0.52 = $860,000
The actual sales data for the month is given below:
Wireless keyboards Wireless mouses Wireless headphones Total

$320,000 $400,000 $280,000 $1,000,000

Required:
Compute the breakeven point of Modern Digital World company based on the actual sales. Explain the
reason of difference (if any) between the break-even point computed on the basis of budgeted sale and
the break-even point computed on the basis of actual sales data.
Solution:
Before computing break-even point based on the actual sales, we need to prepare an income
statement based on the actual sales.
Wireless Wireless Wireless
Total
keyboards mouses headphones

Sales $320,000 100% $400,000 100% 280,000 100% 1,000,000 100%

Less variable expenses 96,000 30% 320,000 80% 154,000 55% 570,000 57%

——– —– ——– —– ——– —– ——– —–

Contribution margin $224,000 70% $80,000 20% $126,000 %45 430,000 43%

——– —– ——– —– ——– —– —–

Less fixed expenses 447,200

——–

Net operating loss (17,200)

——–

Break-even point:
Fixed expenses / CM ratio
447,200 / 0.43
$1,040,000
The reason of difference in break-even point in dollar sales:
The difference in break-even point is because of shift in sales mix.
Percentage of total sales (Budgeted) Percentage of total sales (Actual)

Wireless keyboards 480,000 /1,000,000 = 0.48 or 48% 320,000 /1,000,000 = 0.32 or 32%

Wireless mouse 200,000 /1,000,000 = 0.20 or 20% 400,000 /1,000,000 = 0.40 or 40%

Wireless headphones 320,000 /1,000,000 = 0.32 or 32% 280,000 /1,000,000 = 0.28 or 28%
A shift in sales mix from the products generating high contribution margin to the products generating low
contribution margin decreased the overall contribution margin ratio of the company from 52% to 43% and
increased the dollar sales required to break-even from $860,000 to $1,040,000.
Problem-4 (CM ratio, degree of operating leverage, break-even
point)
ECG company sells lightweight tables. One table is sold for $45. Variable and fixed cost data is given
below:
Variable cost $18 per unit

Fixed cost $540,000 per year

Required:
1. Calculate contribution margin ratio (CM ratio) of the product.
2. Calculate break-even point in dollars using CM ratio.
3. Calculate the increase in net operating income if sales increase by $135,000. Use CM ratio for your
answer.
4. During the last year, ECG company sold 24,000 lightweight tables.
(a). Compute the degree of operating leverage at the last year’s level of sales.
(b). If ECG company manages to increase the sales by 15% next year, how much should net operating
income increase?
Solution:
(1) Calculation of CM ratio:
Contribution margin / Sales revenue
$27* / $45
0.60 or 60%
*$45 – $18 = $27
(2) Calculation of break-even point:
Break-even point in sales = Fixed cost / CM ratio
$540,000 / 0.60
$900,000
(3) Increase in net operating income:
$135,000 × 0.6
$81,000
(4) Degree of operating leverage:
(a).
Degree of operating leverage = Contribution margin / Net operating income
= $648,000* / $108,000**
=6
*[(24,000 × $45) – (24,000 × $18)]
= $1,080,000 – 432,000
= $648,000
**$648,000 – 540,000
= 108,000
(b).
The degree of operating leverage is 6. If sales increase by 15% the net operating income will increase by
90% (15% × 6).
Problem-5 (Change in sales mix, break-even analysis and margin of
safety)
Metro International manufactures two products – plasma TV and high quality laptop. Plasma TV sells for
$800 and high quality laptop for $1200. Company sells its products through its own stores and other
outlets. Total fixed expenses of Metro International are $132,000 per month. Variable expenses and
monthly sales data are given below:
Plasma TV Laptop

Variable expenses per unit $480 $240

Monthly sales in units 200 Units 80 Units

Required:
1. Prepare a contribution margin format income statement showing dollars and percent columns for products
and for the company as a whole.
2. Compute the break-even point in dollars and margin of safety.
3. Metro International is considering to manufacture another product – an inverter. The addition of new
product will not effect the fixed cost of the company. The variable expenses to manufacture and sell an
inverter will be $1,200. If the new product is sold for $1,600 the monthly expected sales are 40 inverters.
(a). Prepare a new contribution margin income statement.
(b). Compute the new break-even point and margin of safety of the company.
4. The president is unable to understand the increase in break-even sales because the new product has
increased the sales revenue and contribution margin without any increase in fixed costs. Explain to the
president the reason of increase in break-even sales.
Solution:
(1) Contribution margin format income statement:
Plasma TV Laptop Total

Sales 160,000 100% 96,000 100% 256,000 100%

Less variable expenses 96,000 60% 19,200 20% 115200 45%

——– —– ——– —– ——– —–

64,000 40% 76,800 80% 140,800 55%

——– —– ——– —– —–

Less fixed expenses 132,000


——–

Net operating income 8,800

——–

(2) Computation of break-even point (BEP) and margin of safety (MOS):


BEP = Total fixed cost / CM ratio
$132,000 / 0.55
$240,000
The break-even point of Metro international in dollars is $240,000. At this point, the company will neither
earn any profit nor suffer any loss.
MOS = Total sales – Break-even sales
$256,000 – 240,000
$16,000
or
16,000 / 256,000 = 0.0625 or 6.25%
The margin of safety is $16,000. It means that if the sales are reduced by $16,000, the Metro
International will break-even.
(3). Addition of new product – inverter:
(a). Contribution margin income statement:
Plasma TV Laptop Inverter Total

Sales 160,000 100% 96,000 100% 64,000 100% 320,000 100%

Less variable expenses 96,000 60% 19,200 20% 48,000 75% 163,200 51%

——– —– ——– —– ——– —– ——– —–

64,000 40% 76,800 80% 16,000 25% 156,800 49%

——– —– ——– —– ——– —– —–

Less fixed expenses 132,000

——–

Net operating income 24,800

——–

(b). Break-even point and margin of safety after the addition of new product:
BEP = 132,000 / 0.49
$269,388
MOS = $320,000 – $269,388
$50,612
(4). Explanation to the president:
The reason of increase in break-even point is the change in sales mix (introduction of new product –
inverter). In spite of the fact that it has increased the sales revenue and total contribution margin, it has
reduced overall CM ratio of the company from 55% to 49%. The reduction in overall CM ratio has
increased break-even point of Metro International.

You might also like